LSAT and Law School Admissions Forum

Get expert LSAT preparation and law school admissions advice from PowerScore Test Preparation.

 Administrator
PowerScore Staff
  • PowerScore Staff
  • Posts: 8917
  • Joined: Feb 02, 2011
|
#35338
Complete Question Explanation

Justify the Conclusion. The correct answer choice is (D)

This social scientist uses some sophisticated language to describe a pretty basic argument: Marxism
describes the movement toward socializing means of production, so it should be considered a
scientific theory. Therefore, the speaker concludes, it is wrong for some to construe it as a political
program whose intent is to radically change society.

The social scientist’s argument breaks down as follows:
  • Premise: Marxism describes the historical movement toward socializing the means of
    production.

    Sub-conclusion: Marxism should thus be considered a scientific theory.

    Conclusion: Thus it is wrong to construe Marxism as a political program aimed at
    changing society.
The stimulus is followed by a Justify the Conclusion question, so the correct answer choice will link
the rogue elements of the argument; the correct answer choice will, when added to the premises of
the stimulus, ensure that the social scientist’s conclusion is properly drawn.
Looking at the conclusion above, it becomes clear that the author believes that if something is
properly regarded as a scientific theory, it is wrong to construe it as a political program aimed at
societal change.

Answer choice (A): The author’s conclusion deals with the issue of whether Marxism can be
properly regarded as a political program. This choice does not link the rogue elements of the
argument and thus does not justify the conclusion from the stimulus.

Answer choice (B): This choice provides that there is an incompatibility between the aims of science
and those seeking political transformation. This does not justify the author’s conclusion, which is
based on the implied notion that scientific theories cannot be properly regarded as political programs.

Answer choice (C): This choice deals with scientific theories but does not link them in any way to
political programs, as is necessary to justify the author’s conclusion. Since this choice fails to fill
in the gaps in the author’s argument, it cannot be the correct answer to this Justify the Conclusion
question.

Answer choice (D): This is the correct answer choice, as it provides that scientific theories cannot
be construed as political programs. Linking this with the premises from the stimulus, the conclusion
becomes clearly justified:
  • Premise: Marxism is properly regarded as a scientific theory.

    Premise: If something is a scientific theory it cannot be considered a political program.

    Justified conclusion: Thus, those who call Marxism a political program are misconstruing it.
Answer choice (E): The stimulus’ conclusion deals with whether Marxism, a scientific theory
according to the author, can properly be termed a political program. This choice deals with the
inevitability of the socialization of the means of production but does not link the rogue elements
from the author’s argument, so it does not justify the author’s conclusion and should be eliminated
from contention.
User avatar
 LSAT2HARD
  • Posts: 14
  • Joined: Jan 03, 2021
|
#87926
If it is a necessary assumption question, will B be a good contender?
 Robert Carroll
PowerScore Staff
  • PowerScore Staff
  • Posts: 1787
  • Joined: Dec 06, 2013
|
#88038
LSAT,

It can be difficult to make judgments about what would be different if the question type were different, but I think in this situation I would not like answer choice (B) for an Assumption question. The conclusion is about how others should regard Marxism. Answer choice (B) claims that certain kinds of aims are mutually incompatible in general. In the case of Marxists, I think it's fair to say the author would think that one characterization of their aims is right and one wrong. But are scientific and political aims in general incompatible, from the author's perspective? We can't be sure.

Robert Carroll

Get the most out of your LSAT Prep Plus subscription.

Analyze and track your performance with our Testing and Analytics Package.